4

The question If I create a varying electric field and it will then create a varying magnetic field, so will it also create light? Will I see a light ray? got me thinking.

I'm pretty sure that if I could put a strong enough magnetic or electrostatic dipole on a shaft spinning sufficiently fast, I could make a low frequency radio wave that would propagate to the far field and receive it with a suitably low frequency antenna and radio receiver.

I'm curious if such a demonstration has actually been done like that.

I'm not asking for analogous demonstrations or "that's in effect what a radio transmitter does with a loop antenna" type answers, I'd like to know if such a practical demonstration has ever been successfully carried out.

There's got to be a real, mechanical rotating shaft and a real magnetostatic or electrostatic dipole, like a bar magnet or two charged spheres separated by an insulating rod for example, and an actual receiver in the far field recording propagating electromagnetic waves, not just some evanescent tail.

Qmechanic
  • 201,751
uhoh
  • 6,308
  • 1
    see, for example, https://ieeexplore.ieee.org/document/8939554 and its references, also https://en.wikipedia.org/wiki/Alexanderson_alternator – hyportnex May 16 '22 at 21:24
  • 1
    @hyportnex oh that's excellent! In fact I'm going to broaden my question to include rotating magnetic dipoles as well, something I should have thought about in the beginning. – uhoh May 16 '22 at 21:29
  • 1
    magnetic dipole is "better" than electric dipole for the former can have higher energy density; higher current at lower voltage is preferable than higher voltage at lower current in a high power transmitter because of arcing and some such. – hyportnex May 16 '22 at 21:31
  • 1
    @hyportnex okay, as long as it's an actual demonstration and not just a proposal; for the purposes of this question there's gotta be a real receiver in the far field picking up real, propagating EM waves. – uhoh May 16 '22 at 21:34
  • 1
    @hyportnex The Alexanderson alternator used a conventional antenna to radiate the energy. – John Doty May 16 '22 at 22:10
  • @JohnDoty that is true, just thought was an interesting historical tidbit to remember a great engineer involved in low frequency EM transmission using a mechanical signal generator ... – hyportnex May 16 '22 at 22:15
  • What about an (elementary) electric monopole moving in a sinusoidal trajectory instead of a circle? If this qualifies, then undulator could be an answer. – Ruslan Aug 27 '22 at 10:27
  • similar question https://physics.stackexchange.com/questions/229366/does-waving-a-magnet-around-create-light – Markoul11 Aug 27 '22 at 11:00
  • Can you be more specific on the term you use "spin"? Do you mean rotate the magnet on an orbital motion around a point in space or spin the magnet on its on N-S axis? – Markoul11 Aug 27 '22 at 11:09
  • @Markoul11 I'm not specifying, and those are not the only two choices of course. And more importantly I'm asking what has been demonstrated and not asking "Would X work?" – uhoh Aug 27 '22 at 13:18
  • https://www.youtube.com/watch?v=V5FyFvgxUhE – Markoul11 Aug 27 '22 at 16:39
  • @Markoul11 Thanks, I'd never seen nor heard of that effect before!! Video links to H. Ucar (2021) Polarity Free Magnetic Repulsion and Magnetic Bound State – uhoh Aug 27 '22 at 22:00
  • 1
    Ucar's research can be also found here: https://www.researchgate.net/profile/Hamdi-Ucar and on his youtube channel: https://www.youtube.com/watch?v=IpYuy5OkXtQ – Markoul11 Aug 28 '22 at 11:30
  • Ucar's demonstations are electromagnetic wave phenomena experiments of VLF radiation. These are not purererly magnetic in nature since he used rotatating magnets on an axis perpendicular to the the N-S axis of magnets. Therefore a changing polarity magnetic field that induces also a charnging electric electric field in space thus electromagnetic waves. Combining diferent phases of EM waves relative fast changing, in combination with the mass inertia of a targer magnet, the magnet is trapped in place levitating. – Markoul11 Aug 29 '22 at 12:26

2 Answers2

1

It's very difficult. A "turnstile" antenna is effectively a rotating dipole, simulated electrically. For the usual half-wave turnstile, the tips of the effective spinning element are moving faster than the speed of light! If you make it smaller, thus reducing the effective linear rotation speed, its capability as a radiator declines rapidly. I don't know of a mechanical system that can rotate >1/100,000 the speed of light.

Edit: In response to comments, here's a quick and dirty engineering sketch of an experiment.

Reference Data for Engineers (E. C. Jordan Ed., 1989) tells me that the radiation resistance of a dipole antenna scales as length squared (Jackson, of course, tells me the same). The current is charge/time, so it scales as velocity for the same charge. Power is proportional to the square of the current times the resistance. So, rotating a dipole at 10^-5 c radiates ~10^-20 of the power that the turnstile radiates for the same amount of charge on the elements. RDfE tells me the natural noise on Earth at 10 kHz (unlikely to be practical as a mechanical rotation speed) is ~160 dB above the nominal thermal.

Nominal thermal is -204 dBW/Hz, thus natural noise at this frequency is -44 dBW. Let's imagine that we can transmit +30 dbW (1 kW) with our turnstile: then, our mechanical version would transmit -170 dBW. Even if our receiver could capture this all (impractical), our SNR is -126dB in a 1 Hz bandwidth. Thus, we'd need to integrate for ~10^13 seconds to detect a signal. I don't expect to live that long ツ

John Doty
  • 20,827
  • 1
    "I'm not asking for analogous demonstrations or 'that's in effect what a radio transmitter does with a loop antenna' type answers..." The whole point of the question is to focus on mechanical rotation. – uhoh May 16 '22 at 22:24
  • @uhoh Yes I know, but the analogous demonstration shows how difficult it is to do what you ask. – John Doty May 16 '22 at 22:45
  • 1
    If it were easy demonstrations would be well known and I wouldn't have asked. If it rotates slower than the speed of light it will produce a strong evanescent field because the induced field (magnetic in the case of rotating charge dipole) will be weak, and while the propagating EM field will be weak, it won't be zero. – uhoh May 16 '22 at 22:56
  • @uhoh Difficult experiments tend to be well-known if they produce a positive result. – John Doty May 16 '22 at 23:01
  • 1
    your answer is "I don't know, but sounds hard"? "well known" by who? Let's see what others post. There's already a proposed device mentioned in a comment under my question which suggests it's doable; not all physicists keep track of what electrical engineers are doing; let's wait and see what other answers are posted. – uhoh May 16 '22 at 23:03
  • @uhoh I can't prove a negative. But it's not just hard, it's practically impossible. – John Doty May 16 '22 at 23:06
  • 1
    That sounds like something that could be demonstrated quantitatively using math and physics rather than just asserting in prose. Can you demonstrate that in a convincing way rather than just saying it without support? I mean, "it's practically impossible because otherwise I would have heard of it" is not really a Physics SE answer. – uhoh May 16 '22 at 23:08
  • @uhoh As I commented, the Alexanderson alternator used a conventional antenna to radiate the energy, so it is not an example of what you seek. Alternators can be very small relative to the wavelength of the waves they make. Transmitting antennas cannot. Receiving antennas are a different game because (classically) you can sense the field without transfer of energy. – John Doty May 16 '22 at 23:14
  • 1
    I'm referring to the other link in the comment. "The proposed mechatronic antenna..." I don't have access to the whole paper, I can't tell if there is a conventional antenna on top of that or not. I'll check the paper later today. Anyway, I'm still looking forward to either a citation of a demonstration or a physics-based (numbers and all) answer to why this is impossible, say based on a quarter-wave receiving antenna and $k_B T$ arguments, which I'm pretty sure it is not. – uhoh May 16 '22 at 23:15
  • @uhoh At 3 kHz the wavelength is 100 km, so those devices are not propagating to the far field. Thus, they also don't answer your question. – John Doty May 16 '22 at 23:18
  • 1
    They may not be using the weak far-field propagation and so may not answer, but we should refrain from calling non-zero as zero. Instead let's use physics and math and build convincing results, something beyond "can't happen because I haven't heard of it". One might start with $v/c$ or $v^2/c^2$ and $k_B T$ arguments for the transmitter and receiver respectively. – uhoh May 16 '22 at 23:20
  • 1
    if all of that is put into an answer, then folks can vote on it and verify it. As a comment it can be deleted at any time and can't be downvoted or easily vetted. – uhoh May 17 '22 at 01:32
  • @uhoh "If one does not have an actual answer to the question as asked, one should not post an answer to a different question, then argue that the OP "didn't ask the question they wanted to".___ Sorry, but you have also replied in a comment with quote: "Then why choose such a clunky example (your question: ...charge spun on a rotating shaft) for which one already knows that it is "practically impossible?". I don't know? You tell me! So I commented then, if you wanted to re-edit your question for clarity of what the argument you ask really is? – Markoul11 Aug 31 '22 at 12:07
  • @Markoul11 if the question seems unclear, why not first leave a comment asking for clarification? – uhoh Aug 31 '22 at 12:18
  • @uhoh "if the question seems unclear, why not first leave a comment asking for clarification?" But, I did! With my comment here!: "EM waves propagate to infinity even inside noise. It then becomes a problem of filtering out the noise and having an enough sensitive receiver. I don't know what you are debating but it seems to me that you are not asking the question you really want to ask here?" – Markoul11 Aug 31 '22 at 13:52
  • @Markoul11 Your comment is irrelevant because I demonstrated the impracticality of adequate noise filtering in my answer above. Of course, uhoh hasn't accepted it. I don't know why. – John Doty Aug 31 '22 at 14:17
  • @JohnDoty Interesting SNR analysis but uhoh seems to insist to a precise answer to his/her question. Therefore there was no need for any answer in this question thread. A comment saying "No." would suffice. Nevertheless, I've found uhoh's question interesting upvotred it and answred it as good as I could, spending a couple of hours in the process searching references. I don't care about any bounty and care more about sharing information and learning something new in the process, but being downvoted for no substantial reason I find discouraging. – Markoul11 Aug 31 '22 at 15:40
-1

Physically rotating the magnet on an orbital motion around a point in space is the principle used Faraday induction, to generate electric current in electric power plants. The electric current is induced by the rotating magnetic field into electric coils.

"The RPM will depend on the number of poles in the alternator. In case of 2 magnetic poles the RPM would be 3600, for 4 pole 1800 for 6 pole 1200 and so on The formula is RPM=120 x f/n Where f is the required frequency and n is the no of poles in the alternator."

https://www.quora.com/In-order-to-generate-electricity-at-a-frequency-of-60Hz-a-generator-in-a-power-plant-must-be-operated-at-how-much-RPM

If you mean in your question just spinning very fast a permanent magnet around its own N-S axis, I don't think you will accomplish anything the field will remain still static and there will be no noticeable EM radiation unless the magnet has large dimensions. The physically spinning magnet will additionally precess that would generate EM waves of $ω_{p}$/2π frequency in Ηz units:

$$ \omega_{\mathrm{p}}=\frac{m g r}{I_{\mathrm{s}} \omega_{\mathrm{s}}} $$

where $ω_{s}$ the spin angular velocity in rad/s, $I_{s}$ the moment of inertia, m the mass of the magnet and g Earth's gravitational acceleration, $r$ the cross-section diameter of the magnet pole divided by two and $ω_{p}$ the EM precession frequency.

To get a feeling of the above equation where Is=7.5x10^-10 Kg m^2 is the calculated moment of inertia of a sphere ferrite magnet, ωs=47.77Hz, m=0.3gr, g=9.81, m/s^2 and r=2.5mm the radius of the sphere magnet. The value obtained of $ω_{p}=5.2$ Hz units radiated EM waves corresponds to a value of no less than 312 rpm, Newtonian precession rotations. Imagine now try to spin on its N-S axis a large 10 cm magnet at 312 rpm! The mechanical stress would break the magnet apart.

If you mean to spin the magnet on an axis perpendicular to its N-S axis then yes it will radiate EM waves of frequency proportional to its spin rpm.

As for related experiments of what you are asking please see my comments on your question.

Here is an example of a rotating magnet ELF-ULF radio transmission antenna, and reception of the EM waves in the near field: https://www.jpier.org/PIERM/pierm72/14.18070204.pd

Reception in the far field is practically very difficult to be demonstrated remotely (i.e. receiver must be at least 2 wavelengths away from transmmiter) for such low frequency EM waves which can have wavelengths of many Km. Unless, you have access to a ULF reception antenna array station (300Hz-3KHz) (wavelengths 1Km to 100Km)! en.wikipedia.org/wiki/Project_Sanguine or maybe HAARP https://en.wikipedia.org/wiki/High-frequency_Active_Auroral_Research_Program .

Of course one could try far reception with a sensitive ULF-ELF electrical short antenna like these magnetic antennas here: aaronia-shop.com/products/antennas-sensors/magnetic-antenna

Otherwise, as far as I know, there is no demonstration or reference currently to be found of such thing you are asking thus if someone has demonstrated reception specific in the far field of ULF or ELF EM waves generated by a mechanically rotating EM charge.

Markoul11
  • 3,853
  • I've asked specifically "Has anyone ever put a magnetic or electrostatic dipole on a rotating shaft, spun it and demonstrated reception of a propagating wave in the far-field?" – uhoh Aug 27 '22 at 13:16
  • @uhoh My answer was updated with a last paragraph. – Markoul11 Aug 29 '22 at 12:31
  • Your last paragraph is an invitation for each reader to go sorting through a long trail of comments (which could at any time in the future be cleaned up) and to look for several of yours. While all of this is interesting, it's not a Stack Exchange answer to the question as asked. I don't want to sound unappreciative, but the question to be answered is "Has anyone X and demonstrated Y?" Of course if it were not so challenging it would have been done a long time ago. I don't see why one would consider rotating a physical dipole "around" it's axis, one would rotate perpendicular to it to radiate. – uhoh Aug 29 '22 at 22:05
  • If links in one of your comments point to an actual demonstration of rotating a magnetic or electric dipole and far field propagating EM radiation was demonstrated, then that certainly can be the basis of an answer. Yes an electric field would be produced in the process, but the actual physical thing that was being physically rotated would be a physical dipole magnet, not a distribution of electric charges. – uhoh Aug 29 '22 at 22:09
  • @utah Magnetism of electrons is actually electromagnetism. There is really nothing else more fundamental :) – Markoul11 Aug 29 '22 at 23:54
  • Okay, so "Has anyone ever put a magnetic or electrostatic dipole on a rotating shaft, spun it and demonstrated reception of a propagating wave in the far-field?" – uhoh Aug 30 '22 at 04:26
  • @uhoh You could. Start mechanically spinning very fast a relative large magnet (bar magnet at least 10cm long) on an axis perpendicular to is N-S magnetization axis and receive the generated EM waves with a VLF radio receiver. Here is an example of a rotating magnet ELF-ULF radio transmission antenna, I hope this funally answers your question :) I have amended my answer with this link: https://www.jpier.org/PIERM/pierm72/14.18070204.pdf – Markoul11 Aug 30 '22 at 14:50
  • " I hope this finally answers your question :)" almost, but not yet: "In Section 4, a prototype is manufactured and measured in the near field." and "which is mainly carried by the non-radiative magnetic field in the near-field." whereas I've asked "...reception of a propagating wave in the far-field?" – uhoh Aug 30 '22 at 15:09
  • @uhoh This is practically no possible to be demonstrated remotely in the far-field (at least 2 wavelengths away) by the authors of this paper unless you have access to a ULF reception antenna array station (300Hz-3KHz) (wavelengths 1Km to 100Km)! https://en.wikipedia.org/wiki/Project_Sanguine or maybe HAARP https://en.wikipedia.org/wiki/High-frequency_Active_Auroral_Research_Program. Of course you could try far reception with a sensitive electrical short antenna like these magnetic antennas here: https://aaronia-shop.com/products/antennas-sensors/magnetic-antenna – Markoul11 Aug 30 '22 at 15:46
  • Then why choose such a clunky example for which one already knows that it is "practically impossible"? If I were to try, I'd excite a current in a small, micro-fabricated 1 to 100 um superconducting loop, or charge a similar-sized electrostatic dipole, suspend it somehow in a vacuum where it could be spun up to much, much higher rates. cf. Has "GHz-spinning dust" ever been demonstrated in the laboratory? – uhoh Aug 30 '22 at 17:09
  • @uhoh EM waves propagate to infinity even inside noise. It then becomes a problem of filtering out the noise and having an enough sensitive receiver. I don't know what you are debating but it seems to me that you are not asking the question you really want to ask here? – Markoul11 Aug 30 '22 at 17:26
  • What I'd really like to ask is "Has anyone ever put a magnetic or electrostatic dipole on a rotating shaft, spun it and demonstrated reception of a propagating wave in the far-field?" and after checking it seems that is exactly the question I did ask! You didn't (even attempt to) write an answer to the question as asked. It's that simple. If one does not have an actual answer to the question as asked, one should not post an answer to a different question, then argue that the OP "didn't ask the question they wanted to". – uhoh Aug 30 '22 at 22:53
  • @uhoh I hardly see how I did not directly answer your question?! You have been given spercific reference and explanations herein to existing research that demonstrated positively that what was asked , in the near reception field and also why this is difficult to be demonstrated in the far reception field and suggested a way to circumvent this problem using an electric short reception antenna. As for the rest of my answer, PSE answers are not restricted by the question and can give any other related background information in order to be complete and canonical for all the readers of PSE. – Markoul11 Aug 31 '22 at 11:23
  • I believe you have give(n)... other related background information... for all the readers of PSE" nicely. But the OP asks if it has actually been done, not "what are some thoughts about the challenges to..." I appreciate all you've written, I don't think you have much more to add so probably no need for further comment exchanges. – uhoh Aug 31 '22 at 12:23